A box contains cards that are numbered from 1 to 100. What is the probability of randomly selecting a number that is less than 12?
1 / 100
1 /12
11 / 100
12 / 100

Answers

Answer 1

Answer:

1/100

Step-by-step explanation:

The box contains 100 cards numbered from 1-100

Since there are 100 cards and one of each number, the probability of pulling 12 is 1/100 like any other card.

Hope it helps!

Answer 2

Answer:

1/100

Step-by-step explanation:

The box contains 100 cards numbered from 1-100

Since there are 100 cards and one of each number, the probability of pulling 12 is 1/100 like any other card.

Hope it helps!


Related Questions

please help me please please please please​

Answers

Answer: [tex]2\frac{2}{3}[/tex]

Step-by-step explanation:

[tex]-2\frac{1}{3}-(-5)=[/tex]

Convert the mixed number to improper fraction. You can do this by multiplying the whole number times the denominator and adding the numerator; over the same denominator.

[tex]-\frac{2*3+1}{3} -(-5)=[/tex]

Solve;

[tex]-\frac{6+1}{3} -(-5)=[/tex]

[tex]-\frac{7}{3} -(-5)=[/tex]

Now, get rid of the two negatives by multiplying.

[tex]-\frac{7}{3} +5=[/tex]

Remember 5 has a 1 below; therefore, we can subtract these two fractions by getting both to a same denominator. Let's rewrite 5 as a fraction.

[tex]-\frac{7}{3}+\frac{5}{1} =[/tex]

To be able to convert 1 to 3, we have to multiply by 3. Remember that if you change the denominator, you have to change the numerator as well.

[tex]-\frac{7}{3}+\frac{5}{1} (\frac{3}{3} ) =[/tex]

Solve;

[tex]-\frac{7}{3}+\frac{15}{3} =[/tex]

Subtract numerators and keep the same denominator.

[tex]\frac{-7+15}{3}=\frac{8}{3}[/tex]

Convert to a mixed number. To do this, first, divide:

8/3=2

6

----

2

The 2 after the equal sign is the whole number, the two in the remainder is the numerator and the 3 is the denominator. Leaving our result as: [tex]2\frac{2}{3}[/tex]

find the missing letter N if the width is 2ft and the area equal 28 sq ft​

Answers

Answer:

Step-by-step explanation:

Area = 28 Sq.ft

length * width = 28

2 * n = 28

Divide both sides by 2

2n/2 = 28/2

n = 14 ft

What is 1/2 of 7/8? In fraction firm

Answers

1/2*7/8=7/16

Answer:

7/16

The required fraction of 1/2 of 7/8 is given as 7/16.

What is the fraction?

Fraction is defined as the number of compositions that constitutes the Whole.

What is simplification?

The process in mathematics to operate and interpret the function to make the function or expression simple or more understandable is called simplifying and the process is called simplification.

here,
1/2 of 7/8 = 1/2 × 7/8
               = 7 / 16

Thus, the required fraction of 1/2 of 7/8 is given as 7/16.

Learn more about fractions here:

https://brainly.com/question/10708469

#SPJ2

help please!!!!!!!!!!

Answers

Answer:

[tex]15 × 10^{0}[/tex]

Step-by-step explanation:

5 × 10^-7 × 3 × 10^6

15 × 10^-1

= 1.5 × 10^0

Answer:

15 × 10^{-1} is the correct

Step-by-step explanation:

The diagram below is divided into equal parts. Which shows the ratio of unshaded sections to shaded sections

Answers

Answer:

5 to 1.

Step-by-step explanation:

There are 5 unshaded and one shaded section so it's

5:1.

The ratio of unshaded sections to shaded sections is;

Option D; 5 to 1

We are told that the diagram is divided into equal parts.

Now, from the given, we can see that there are 6 equal parts. However, out of the 6 equal sections, only one section is shaded while 5 sections are unshaded.

This means that the ratio of unshaded sections to shaded sections is; 5 to 1.

In conclusion, the ratio of unshaded sections to shaded sections from the given diagram that was divided into equal parts is; 5 to 1.

Read more at; https://brainly.com/question/20094176

Sondra gets $75 a week, plus 10 percent of sales. Which equation best represents her salary? Group of answer choices

Answers

75 + (10/75)
I do not know for sure

What is the least common multiple of 8, 12 and 16?

Answers

The answer is 48 :)) hope I helped!

Answer:

b

Step-by-step explanation:

Which of the following number lines represents the solution to m/-2 - 7 = -9?

Answers

Answer:

Step-by-step explanation:

Answer: m=1    

Step-by-step explanation:

Please round 0.02623 to the nearest hundred.
||||
Thank you guys! Please answer soon!

Answers

.03 is the answer. hope it help!:-)

Answer:

0.02623 rounds to 0.03

Step-by-step explanation:

ones. tenths hundredths thousands

we look at the thousandths place when rounding to the hundredths

the thousands place is 6 (which is 5 or greater) so we will round the hundreds place up 1

0.02623 rounds to 0.03

Jael and Kelsey work at a bakery. Jael makes 8 cupcakes for every 5 cupcakes that Kelsey
makes. If Kelsey makes 175 cupcakes in one day, what is the total combined number of cupcakes that Kelsey and Jael make in one day?

Answers

Answer:

35

Step-by-step explanation:

175/5

Answer:

455 cupcakes

Step-by-step explanation:

If Jeal makes 8 cupcakes for every 5 cupcakes that Kelsey makes and Kelsey made 175 cupcakes then you divide 175 by 5 and you get 35. Then mulitply 35 by 8 and you will get 280. Lastly you add 175 and 280 and you will get 455.

help quick!!!!?????​

Answers

Omg I’m so sorry to interrupt but I really need help on my homework :( it’s two questions for 20 points||| can someone please help

Answer:

none

Step-by-step explanation:

57+75+95= 227

24+70+75= 169

48+64+80= 192

19+27+80=73

A right triangle equals 180 degrees.

. A kite is in the shape of a parallelogram
with a 4.5 cm base and a height of
7.9 cm. What is the area of the kite?

Answers

Answer:

I believe the answer is 35.55 cm

Step-by-step explanation:

Answer:

35.55 cm

Step-by-step explanation:

To find the answer of this question is to use the method bxh.

  4.5 cm

x 7.9 cm

----------------

  35.55 cm

When you graduate from high school, you deposit all of your graduation cash into an account earning 5% interest. Ten years from now you will have
$5000 in that account. How much money did you deposit?

Answers

If it’s 5% interest per year, then you deposited $2500. If it’s 5% interest total over the total 10 years, then you deposited $4750

Jack divided his fortune of $92,000 into 8 equal parts. He gave 4 parts to his wife, 1 part to his son and divided the remaining 3 parts equally among 5 charities.
How much more money did his son get than each charity?

Answers

Answer:

4,600

Step-by-step explanation:

92,000/8= 11500

11500 x 4= 46000 to his wife

11500 for the son

92000-57500= 34500

34500/5= 6,900 each charity

11500-6900= 4600 is how more he is getting than each charity

Answer:

4600 more money

Step-by-step explanation:

92000 ÷ 8 = 11500

1 part to his son

3(11500)= 34500

34500 ÷ 5 = 6900

11500-6900= 4600

Angles
SU and VX are parallel line.
which angles are alternate exterior angles?
XWY and STR
XWY and VWT
XWY and VWY
XWY and XWT​

Answers

Answer: XWY and STR

I tend to think of parallel lines as train tracks (the metal rail part anyway). Inside the train tracks is the interior region, while outside the train tracks is the exterior region. Alternate exterior angles are found here. Specifically they are angles that are on opposite or alternate sides of the transversal cut.

Both pairs of alternate exterior angles are shown in the diagram below. They are color coded to help show how they pair up and which are congruent.

A thing to notice: choices B, C, and D all have point W as the vertex of the angles. This means that the angles somehow touch or are adjacent in some way due to this shared vertex point. However, alternate exterior angles never touch because parallel lines never do so either. We can rule out choices B,C,D from this reasoning alone. We cannot have both alternate exterior angles on the same exterior side of the train tracks. Both sides must be accounted for.

please help i suck at math:'(

Answers

Answer: B 6400 m

Step-by-step explanation:

the answer is B. 6,400 meters

Please HELP!!!!!!!!!

Answers

Answer:

The sides are 3 and 9

Step-by-step explanation:

Since this is a right triangle, we can use the pythagorean theorem

a^2+b^2 = c^2 where a and b are the legs and c is the hypotenuse

x^2 + (3x)^2 = (3 sqrt(10))^2

x^2 + 9x^2 = 9*10

10x^2 = 90

Divide each side by 10

x^2 = 90/10

x^2 =9

Take the square root of each side

x = sqrt(9)

x = 3

The sides are x=3  and 3x = 3(3) = 9

Answer: x=10

Step-by-step explanation:

Since the original statements were x, 3x, and 3[tex]\sqrt{x}[/tex], is already given as 10 in 3[tex]\sqrt{x}[/tex], therefore, 3x is 3*10 and x is 10

Question 1
2 pts
P is between Jand K. The distance between Jand Pis 7 more than 3 times the distance
between P and K. If JK = 55, what is PK?

Answers

Answer:

29

Step-by-step explanation:

B
The measure of ZDAB equals 110° The
measure of ZACB equals 20°.
What is mZ ABC?
D
A
Enter your answer in the box

Answers

Answer:

Given

¯¯¯¯¯¯¯¯¯

D

A

C

is a straight line, we can say its full measurement is 180°. If we know a segment of this angle,

D

A

B

, is 105°, we can know that the other segment

B

A

C

must equal 75, since

105

+

75

=

180

.

Once we've found the measure of

B

A

C

to be 75°, and given the measure of

A

C

B

to be 35, we can then find the measurement of

A

B

C

.

Since we know that the sum of any triangle's angles is 180°, we can then use these known measurements as I said above.

75

+

35

+

m

A

B

C

=

180

110

+

m

A

B

C

=

180

m

A

B

C

=

70

What is the value of x when 1/3x=9 1/3?
Helppp nowww

Answers

Answer:

x=28

Step-by-step explanation:

[tex]\frac{1}{3} x=9\frac{1}{3} \\\frac{1}{3} x=\frac{28}{3} \\x=28[/tex]

The value of x is 28.

What is mixed fraction?

A mixed fraction represented with its quotient and remainder is a mixed fraction.

How to convert a mixed fraction fraction into an improper fraction?

To convert a mixed fraction into improper fraction first we multiply the denominator of the proper fraction to the whole number attach with it then we add the numerator.

According to the question:

we have

[tex]\frac{1}{3}x =9\frac{1}{3}[/tex]

for finding the value of x

[tex]\frac{1}{3}x =9\frac{1}{3}[/tex]

⇒[tex]\frac{1}{3}x=\frac{28}{3}[/tex]

⇒[tex]\frac{3}{3} x= 3[/tex]×[tex]\frac{28}{3}[/tex]                      (multiply both the sides by 3)

⇒ [tex]x= 28[/tex]

The value of x is 28.

Learn more about for finding the value of x here:https://brainly.in/question/11504807

#SPJ2

Please help with this

Answers

Answer:

[tex]\frac{\sqrt{8} }{\sqrt{10} }[/tex]

Step-by-step explanation:

A perfect square is a number that has two identical factors, meaning it can be square rooted. The numbers 64 and 100 are an example:

8 · 8 = 64

10 · 10 = 100

So when you take the square root of [tex]\frac{64}{100}[/tex], you get:

[tex]\frac{\sqrt{8} }{\sqrt{10} }[/tex]

It doesn't say to simplify, so we can just leave it at that.

20 points if you can help me with problem 2.

Answers

I think it’s 9 in the 3rd lane the 3 rd one

Answer:

3, 4, 5, 6, 7, 9, 10, 11

Step-by-step explanation:

Actually it's 10 points but whatever:

I number the figures like this:

1, 2, 3

4, 5, 6

7, 8, 9

3, 4, 5, 6, 7, 9, 10, 11 work because if you cut them out and glued the sides correctly, it will become a cube.

Identify the equation or inequality of the graph

A)2x-3y=6
B)3x+2y=-6
C)2x-3y=-6
D)3x-2y=6

Answers

Answer:b

Step-by-step explanation:

Answer:

D

Step-by-step explanation:

Need Help ASAP it's due today. Angelica uses the point (4,3) to represent the location of her house and uses the point (10,8) to represent the location of a gas station. Each unit on the graph represents 1 mi. How far is the gas station from Angelica's house? SHOW YOUR WORK. ( The teacher said to Plot a third point, connect the third point to make a right triangle, find the lengths of the legs of the triangle and then use the pythagorean theorem to find the distance between the gas station and Angelica's house). ​Only answer if you truly know how to solve this please. Will Mark Brainliest

Answers

Answer:

About 7.8 miles.

Step-by-step explanation:

We can use the distance formula to find the distance from the gas station to the house.

However, your teacher said to use the Pythagorean Theorem to find the distance.

Going by your teacher's preferred method:

(4,3), (10,8), and (10,3) would make a right triangle.

[See Picture]

Angelica's house is at point (4,3). It is 6 units from our new third point.

The gas station is at point (10,8). It is 5 units form out new third point.

The legs are 5 and 6 units. The distance from the house to the gas station would be our hypotenuse.

Use the Pythagorean Theorem to find the length of the hypotenuse.

[tex]a^2 + b^2 = c^2\\\\\text {Replace a and b with the appropriate values: }\\6^2 + 5^2 = c^2\\\text {Solve: }\\6^2 + 5^2 = c^2\\6^2 = 36\\5^2 = 25\\\rightarrow 36 + 25 =c^2\\36 + 25 = 61\\\rightarrow 61=c^2\\\sqrt{61}=\sqrt{c} \\\boxed {7.81024967591=c}[/tex]

7.81024967591 ≈ 7.8

The hypotenuse is about 7.8.

Therefore, the distance between the gas station and Angelica's house should be about 7.8 miles.

Brainilest Appreciated!

Answer:

AG = 7,8

Step-by-step explanation:

Plot the third point at (10,3) and let's call it point B.

Now construct the right sided triangle A B G

With A = (4,3) , B = (10,3) and G = (10,8).

AB is horizontal so subtract the x coordinates of point A and point B. So AB has length 10 - 4 = 6

BG is vertical so subtract the y coordinates of point B and point G. So BG has length 8 - 3 = 5

Now use pythagorean theorem to calculate AG

In any right sided triangle you can use a² + b² = c²

In our case AB² + BG² = AG²

Substitute AB = 6 and BG = 5

6² + 5² = AG²

AG² = 6*6 + 5*5

AG² = 36 + 25

AG = + - SQRT(61)

{Only the + variant of the SQRT has a meaning here.}

AG = 7,81 rounded on one decimal AG = 7,8

A data set has a lower quartile of 3 and an interquartile range of 5. Which box plot could represent this data set?
0
5
10
15
20
25
5
10
15
20
25
5
10
15
20
25
5
10
15
20
25
Which box plot could represent this data?

Answers

Answer:

First box plot

Step-by-step explanation:

In the figure attached, the options are shown.

The left end of the box is the lower quartile (also called quartile 1 or Q1)

The right end of the box is the upper quartile (also called quartile 3 or Q3)}

The interquartile range (IQR) is computed as follows:

IQR = Q3 - Q1

If Q1 = 3 and IQR = 5, then Q3 = 5 + 3 = 8

The first box plot is the only one with a lower quartile of 3 and an interquartile range of 5

The box plot that could represent a data set that has an interquartile range of 5 is the box plot shown in the image attached below (see attachment).

What is Interquartile Range (IQR)?

Interquartile range (IQR) of a data set is the distance between the lower quartile (Q1) and the upper quartile (Q3) on a box plot.

Thus, a box plot that has a lower quartile of 3 and interquartile range of 5 will have a distance of 5 from the lower quartile to the upper quartile.

Therefore, the box plot that could represent a data set that has an interquartile range of 5 is the box plot shown in the image attached below (see attachment).

Learn more about interquartile range on:

https://brainly.com/question/4102829

TeeVee Electronics, Inc., makes console and wide-

screen televisions. The equipment in the factory allows +

for making at most 450 console televisions and 200

wide-screen televisions in one month. It costs $600 per

unit to make a console television and $900 per unit to

make a wide screen television. The profit on each

console television is $125 per unit, while the profit for

each wide screen television is $200 per unit. During the -

month of November, the company can spend $360,000 to make these televisions. To

maximize profits, how many of each type should they make?

LET

Answers

Answer:

First we need to put all the given information in a table, that way we'll express it better into inequalities.

                                     Cost      Production Max.

Console screen (x)      $600              450

Wide-screen (y)           $900              200

                                 $360,000

We have:

[tex]600x+900y \leq 360,000[/tex]

Because they can't spend more than $360,000 in production.

[tex]x\leq 450\\y\leq 200[/tex]

Because the number of television is restricted.

The profit function is

[tex]P(x,y)=125x+200y[/tex] (this is the function we need to maximize).

First, we need to draw each inequality. The image attached shows the region of solution, which has vertices (0,200), (300,200), (450, 100) and (450,0).

Now, we test each point in the profit function to see which one gives the highest profit.

For (300,200):

[tex]P(300,200)=125(300)+200(200)=37,500+40,000=77,500[/tex]

300 console screen and 200 wide screen give a profit of $77,500.

For (450,100):

[tex]P(450,100)=125(450)+200(100)=56,250+20,000=76,250[/tex]

450 console screen and 100 wide screen give a profit of $76,250.

Therefore, to reach the maximum profits, TeeVee Electronic, Inc., must produce 300 console screen televisions and 200 wide-screen televisions to profit $77,500,

Sara solved the equation -6(x-2) + 3x = -3(x+3) + 21 What is the solution?
pls help :)

Answers

To solve you can use inverse operations.

Here are the steps to find the answer:

1. Write out the equation: -6(x-2)+3x= -3(x+3) +21

2. Distribute both sides: -6x+12+3x= -3x-9+21

3. Simplify both sides: -3x+12=-3x+12

4. This equation is true for all values, or infinity, because the equation is the same on both sides

Let me know if you need further explanation. I hope this helps.

A student is ordering a flower arrangement. She can choose any combination of tulip and carnations for her flower arrangement , and she does not want to spend more than $45. If tulip cost $5 each and carnations cost $3 each, which inequality represents all possible combinations of x tulips and y carnations ? A. 5x + 3y < 45 B. 5x + 3y <= 45 C. 3x + 5y > 45 3x + 5y <= 45
Helppp!

Answers

Answer:

[tex]5x+3x \leq 45[/tex] represents all possible combinations of x tulips and y carnations

Step-by-step explanation:

We are supposed to find inequality represents all possible combinations of x tulips and y carnations

Cost of 1 tulip = $5

Cost of x tulips = 5x

Cost of 1 carnation =$3

Cost of y carnations = 3y

We are given that she does not want to spend more than $45.

So, She can spend less than or equal to 45

So, required inequality :[tex]5x+3x \leq 45[/tex]

So, Option B is true

[tex]5x+3x \leq 45[/tex] represents all possible combinations of x tulips and y carnations

Answer:

it would be 5x + 3y <= 45

Step-by-step explanation:

I literally just did this question on my math homework and I got it right .

Evaluate the expression below for x = 2, y = –3, and z = –1.

A.

–23

B.

–5

C.

13

D.

27

Answers

Correct question:

Evaluate the expression below for x = 2, y = –3, and z = –1.

Given: [tex] x^2 z^2 -y^2(x+z) [/tex]

Options:

A.–23

B.–5

C.13

D.27

Answer:

B. -5

Step-by-step explanation:

In this question we are given the following values:

x = 2

y = -3

z = -1

Given the expression:

[tex] x^2 z^2 -y^2(x+z) [/tex]

To evaluate, let's substitute for 2, -3 & - 1 for x, y & z respectively

Therefore,

[tex] x^2 z^2 -y^2(x+z) [/tex]

Substituting values, we have:

[tex] (2^2) (-1^2) - (-3^2)(2+(-1)) [/tex]

Simplifying, we have:

[tex] [(2^2) (-1^2)] - [(-3^2)(2-1)] [/tex]

[tex]= [(4) (1)] - [(9)(1)] [/tex]

[tex]= 4 - 9[/tex]

= -5

The correct answer is option B

Each big square below represents one whole.
What percent is represented by the shaded area?​

Answers

Answer:

59%

Step-by-step explanation:

No. of squares in a whole = 10*10=100

No. of spaces shaded = 59

No. of spaces not shaded = 41

Hence, percent represented by the shaded area = 59%

Other Questions
When you cough,the radius of your trachea (windpipe) decreases,affecting the speed S of the air in the trachea. If r0 is the normal radius of the trachea, the relationship between the speed S of the air and the radius r of the trachea during a cough is given by a function of the formS(r) = (r0 - r) ar^2where a is positive constant. Find the radius r for which the speed of the air is greatest. Students are voting on the color of T-shirts to wear on their field trip. There are 160 students-65 boys and 95 girls-going on thefield trip. A random sample of students is chosen. Surveying the random sample produced a representative sample of thepopulation. Which was most likely true of the representative sample?VAThe representative sample contained all 160 students,The representative sample contained more boys than girls.The representative sample contained more girls than boys.The representative sample contained an equal number of boys and girls,Save and ExitNextSubmitMark this and retum What led the United States to remain neutral on the Rwandan genocide?A trade agreement with the Tutsis prevented American involvement.British forces agreed to handle the matter eliminating the need for American involvement.The death of US soldiers in Somalia led America to remain neutral.The Geneva Convention prevented international intervention during a civil war. What is 3/4 + 1/3 ?* 15p* whats the constant of the function? Melanie had 4 2/3 pounds of chopped walnuts. She used 1 1/4 pounds in a recipe. How many pounds of chopped walnuts did she have left? An outside supplier has offered to provide the annual requirement of 7,200 of the parts for only $13 each. The company estimates that 60% of the fixed manufacturing overhead cost above could be eliminated if the parts are purchased from the outside supplier. Assume that direct labor is an avoidable cost in this decision. Based on these data, the financial advantage (disadvantage) of purchasing the parts from the outside supplier would be: What gives the US government the power to collect taxes?O the ConstitutionOOOOlaws passed by Congressan executive ordercommon law Explain what the quote means to you or could mean to others. Please reply in 8-10 sentences. Include the quote WITHIN your response. Be sure to demonstrate correct use of sentence variety, spelling, punctuation, comma usage, etc."Come what may, all bad fortune is to be conquered by endurance." Virgil At the beginning of last year, Tarind Corporation budgeted $900,000 of fixed manufacturing overhead and chose a denominator level of activity of 600,000 machine-hours. At the end of the year, Tari's fixed manufacturing overhead budget variance was $12,000 favorable. Its fixed manufacturing overhead volume variance was $19,200 favorable. Actual direct labor-hours for the year were 625,000. What was Tari's total standard machine-hours allowed for last year's output? What type of function best models the data in this table? Justify your reasoning.x258111417y162534435261 x^2 = 8x - 15 ....................... Finding a Linear Regression ModelWarm-UpFind a linear function that models the data in the table2yf(x) =-4-6-1-1012437 Factor the polynomial.125x^3+27 Read the two sentences. I cannot wait to see this movie. The main character is played by my favorite actress. Which sentence expresses the same idea in a complex sentence? 1. To measure the resting heart rate of an animal biologist use the function h(m) = 530m-C), where his resting heart rate in beats per minute, given the mass m in pounds.Part A: What is a feasible domain for the function him? What does Dr.Martin Luther king jr mean when he speaks about a promissory note or checks.A he uses it as a metaphor to represent the equality minorities are awed .B the money he is owed for working daily .C none of the above. The throwing back by a wall or barrier of a sound wave without absorbingit. *1 point French help using the subjunctive.I need help asap describing in french what is going on in this images. 2-3 sentences... Historically, why has Israel-Palestine been important?